Đến nội dung

Hình ảnh

$\left\lfloor\text{Phần nguyên}\right\rfloor$

Floor Function Phần nguyên

  • Please log in to reply
Chủ đề này có 26 trả lời

#21
Peter Pan

Peter Pan

    Sĩ quan

  • Thành viên
  • 360 Bài viết

He he... Mình search trên Wikipedia tìm được 2 bài khá hay
CMR:

  • $\lfloor \sqrt{n}+\sqrt{n+1}\rfloor=\lfloor \sqrt{4n+2}\rfloor$ với mọi $n \in \math\bb{N}$

ngay tại bài này ta có bài toán sau. với n nguyên dường cho trước, tìm các số nguyên dươn x,y sao cho
$\sqrt{n}+\sqrt{n+1}\leq \sqrt{x}+\sqrt{y}\leq \sqrt{4n+2}$
(đề thi HSG 10 TPĐN những năm trước :delta )

\


#22
dark templar

dark templar

    Kael-Invoker

  • Hiệp sỹ
  • 3788 Bài viết

Tính các tổng sau:

$a.\;\;\;\;\;S_n=\sum_{k=0}^n \lfloor\sqrt{k}\rfloor$

$b.\;\;\;\;\;S_n=\sum_{k=0}^n \lfloor\sqrt{2k}\rfloor $

Câu a em chỉ mới có 2 ý tưởng để làm thôi :Rightarrow Nhưng ko biết đúng hay sai nữa .Có gì sai sót mong anh đừng trách :Rightarrow
Ý tưởng đầu tiên của em là kẹp $S_n$ giữa 2 số nguyên có hiệu bằng 2 ,thì khi đó $S_n$ sẽ bằng số nguyên đứng giữa 2 số nguyên đó .Nhưng với ý tưởng này ,em mới chỉ kẹp đc như sau:$n \le S_n < \dfrac{{3n^2 + 2n}}{8}$.Kẹp như vầy thì cũng chả đi tới đâu cả :perp

Còn ý tưởng thứ 2 của em là quy về bài toán "Trong dãy từ 1 đến n,có bao nhiêu số chính phương?" .Cái này thì em chỉ mới làm sơ như sau:
Xét nếu $\left( {k + 1} \right)^2 > n(k \in N)$ thì trong dãy $S_n$ sẽ có k số chính phương .Sau đó ta sẽ tính khoảng cách giữa 2 số chính phương để tất cả phần nguyên của các số trong khoảng đó đều sẽ bằng số chính phương nhỏ hơn->ta sẽ thiết lập đc một tổng ko còn dính đến phần nguyên nữa ->sử dụng dãy số để tính.Trường hợp nhỏ hơn làm tương tự .
Nhưng vấn đề mà ý tưởng này mắc phải đó là có thể n nằm giữa 2 số chính phương liên tiếp ,vậy làm sao ta biết đc n sẽ dừng lại ở số nguyên nằm giữa 2 số chính phương trên?? Ta tính phần "dôi" ra này bằng cách nào?
Đến đây thì em bó tay rồi :Rightarrow
P/s:Nếu đc anh có thể share lời giải của 2 bài trên cho em ko???Thanks anh nhiều :perp
"Do you still... believe in me ?" Sarah Kerrigan asked Jim Raynor - Starcraft II:Heart Of The Swarm.

#23
hxthanh

hxthanh

    Tín đồ $\sum$

  • Hiệp sỹ
  • 3916 Bài viết

Câu a em chỉ mới có 2 ý tưởng để làm thôi :perp Nhưng ko biết đúng hay sai nữa .Có gì sai sót mong anh đừng trách :perp
Ý tưởng đầu tiên của em là kẹp $S_n$ giữa 2 số nguyên có hiệu bằng 2 ,thì khi đó $S_n$ sẽ bằng số nguyên đứng giữa 2 số nguyên đó .Nhưng với ý tưởng này ,em mới chỉ kẹp đc như sau:$n \le S_n < \dfrac{{3n^2 + 2n}}{8}$.Kẹp như vầy thì cũng chả đi tới đâu cả :perp

Còn ý tưởng thứ 2 của em là quy về bài toán "Trong dãy từ 1 đến n,có bao nhiêu số chính phương?" .Cái này thì em chỉ mới làm sơ như sau:
Xét nếu $\left( {k + 1} \right)^2 > n(k \in N)$ thì trong dãy $S_n$ sẽ có k số chính phương .Sau đó ta sẽ tính khoảng cách giữa 2 số chính phương để tất cả phần nguyên của các số trong khoảng đó đều sẽ bằng số chính phương nhỏ hơn->ta sẽ thiết lập đc một tổng ko còn dính đến phần nguyên nữa ->sử dụng dãy số để tính.Trường hợp nhỏ hơn làm tương tự .
Nhưng vấn đề mà ý tưởng này mắc phải đó là có thể n nằm giữa 2 số chính phương liên tiếp ,vậy làm sao ta biết đc n sẽ dừng lại ở số nguyên nằm giữa 2 số chính phương trên?? Ta tính phần "dôi" ra này bằng cách nào?
Đến đây thì em bó tay rồi :perp
P/s:Nếu đc anh có thể share lời giải của 2 bài trên cho em ko???Thanks anh nhiều :Rightarrow

Ý tưởng thứ 2 của em chính là chìa khóa giải quyết 2 bài toán này đấy!

Bài 1:
$S_n=\sum_{k=0}^n \left\lfloor\sqrt{k}\right\rfloor$
Để viết cho gọn ta đặt $\left\lfloor \sqrt n\right\rfloor=m$
Như vậy $m\le \sqrt n< m+1 \Leftrightarrow m^2\le n <(m+1)^2$
(n nằm trong khoảng đấy chứ đâu! :perp )
Trước hết ta có nhận xét sau
Trên mỗi đoạn $[i^2,i^2+2i]$ (giữa hai số chính phương liên tiếp) có $2i+1$ số dạng $i^2+j,\;(0\le j\le 2i)$
Các số này đều thỏa: $\left\lfloor\sqrt{i^2+j}\right\rfloor=i$
Do đó $S_n$ được chia thành các tổng:

$S_n=\sum_{k=0}^{1^2-1} \left\lfloor\sqrt{k}\right\rfloor+\sum_{k=1^2}^{2^2-1} \left\lfloor\sqrt{k}\right\rfloor+...+\sum_{k=(m-1)^2}^{m^2-1} \left\lfloor\sqrt{k}\right\rfloor+\sum_{k=m^2}^n \left\lfloor\sqrt{k}\right\rfloor$

$S_n=\sum_{i=0}^{m-1} \left(\sum_{k=i^2}^{i^2+2i} \left\lfloor\sqrt{k}\right\rfloor\right)+\sum_{k=m^2}^n \left\lfloor\sqrt{k}\right\rfloor$

$S_n=\sum_{i=0}^{m-1} (2i+1)i +\sum_{k=m^2}^n \left\lfloor\sqrt{k}\right\rfloor$
$S_n=\sum_{i=0}^{m-1} \left(\dfrac{2}{3}\left((i+1)^3-i^3\right)-\dfrac{1}{2}\left((i+1)^2-i^2\right)-\dfrac{1}{6}\right)+(n-m^2+1)m$

(viết dưới dạng sai phân nhìn cho dễ :perp)
(Cái tổng dôi ra, gồm $n-m^2+1$ số có giá trị bằng $m$)

$S_n=\dfrac{2m^3}{3}-\dfrac{m^2}{2}-\dfrac{m}{6}+nm-m^3+m$

$\boxed{S_n=\sum_{k=0}^n \left\lfloor\sqrt{k}\right\rfloor=n.m-\dfrac{m(m-1)(2m+5)}{6},\;\;\;\text{voi' }m=\left\lfloor\sqrt{n}\right\rfloor}$
----
Bài kia tương tự nhé! :Rightarrow. (Khó hơn nhiều đấy!:Rightarrow)

Bài viết đã được chỉnh sửa nội dung bởi hxthanh: 07-02-2011 - 22:30


#24
hxthanh

hxthanh

    Tín đồ $\sum$

  • Hiệp sỹ
  • 3916 Bài viết

Tính các tổng sau:

$a.\;\;\;\;\;S_n=\sum_{k=0}^n \lfloor\sqrt{k}\rfloor$

$b.\;\;\;\;\;S_n=\sum_{k=0}^n \lfloor\sqrt{2k}\rfloor $

Gợi ý: Đáp án câu b là

$\boxed{S_n=\sum_{k=0}^n \left\lfloor\sqrt{2k}\right\rfloor=n.m-\dfrac{m(m-1)(2m+5)}{12}+\dfrac{1}{2}\left\lfloor\dfrac{m}{2}\right\rfloor,\;\;\text{voi' }\;m=\left\lfloor\sqrt{2n}\right\rfloor}$

Bài viết đã được chỉnh sửa nội dung bởi hxthanh: 07-02-2011 - 22:31


#25
dark templar

dark templar

    Kael-Invoker

  • Hiệp sỹ
  • 3788 Bài viết
Góp vui 1 bài Phần nguyên +Dãy số cho anh hxthanh
Cho $\left\{ {u_n } \right\}:u_n = \underbrace {\sqrt {1995 + \sqrt {1995 + ... + 1995} } }_{ \times n}.\left\lfloor {u_n } \right\rfloor = ?$(có n dấu căn)
"Do you still... believe in me ?" Sarah Kerrigan asked Jim Raynor - Starcraft II:Heart Of The Swarm.

#26
hxthanh

hxthanh

    Tín đồ $\sum$

  • Hiệp sỹ
  • 3916 Bài viết

Góp vui 1 bài Phần nguyên +Dãy số cho anh hxthanh
Cho $\left\{ {u_n } \right\}:u_n = \underbrace {\sqrt {1995 + \sqrt {1995 + ... + 1995} } }_{ \times n}.\left\lfloor {u_n } \right\rfloor = ?$(có n dấu căn)

Bài này anh chặn giá trị của $\{u_n\}$ như sau:
Theo đề bài, thì
$u_1=\sqrt{1995}; u_{n+1}=\sqrt{1995+u_n};\;\;\;\forall n\ge 1$
Ta có
$\left\lfloor u_1\right\rfloor =\left\lfloor\sqrt{1995}\right\rfloor=44$
Với $n\ge 2$, ta CM rằng
$45<u_n<46\;\;\;(1)$
CM (1) bằng quy nạp
...
Giả sử đúng đến n
Từ biểu thức truy hồi chứng minh (1) đúng với n+1 ...
Kết luận
$\left\lfloor u_n\right\rfloor=45$
-----
( thông cảm anh ngại trình bày quá! :Rightarrow)

#27
dark templar

dark templar

    Kael-Invoker

  • Hiệp sỹ
  • 3788 Bài viết

Bài này anh chặn giá trị của $\{u_n\}$ như sau:
Theo đề bài, thì
$u_1=\sqrt{1995}; u_{n+1}=\sqrt{1995+u_n};\;\;\;\forall n\ge 1$
Ta có
$\left\lfloor u_1\right\rfloor =\left\lfloor\sqrt{1995}\right\rfloor=44$
Với $n\ge 2$, ta CM rằng
$45<u_n<46\;\;\;(1)$
CM (1) bằng quy nạp
...
Giả sử đúng đến n
Từ biểu thức truy hồi chứng minh (1) đúng với n+1 ...
Kết luận
$\left\lfloor u_n\right\rfloor=45$
-----
( thông cảm anh ngại trình bày quá! :Rightarrow)

Anh làm đúng rồi đó :Rightarrow Bài này mang tính mò nhiều hơn là tư duy :Rightarrow
"Do you still... believe in me ?" Sarah Kerrigan asked Jim Raynor - Starcraft II:Heart Of The Swarm.





Được gắn nhãn với một hoặc nhiều trong số những từ khóa sau: Floor Function, Phần nguyên

0 người đang xem chủ đề

0 thành viên, 0 khách, 0 thành viên ẩn danh